Question

.2, Part B: Kirchhoffs law onsider the circuit shown below. R,-100 kΩ I. Before the lab session, use the Kirchhoffs loop rule to manually calculate: a) the current through each resistor, b) the power supplied by cach source, and e) the power dissipated in each resistor. and prepare your report and have it with you electronically at the time of your lab session
0 0
Add a comment Improve this question Transcribed image text
Answer #1

R2 100 000.a R3 20000 2. o kvl to second loop 6 2 R3kindly give rating

Add a comment
Know the answer?
Add Answer to:
.2, Part B: Kirchhoffs law onsider the circuit shown below. R,-100 kΩ I. Before the lab...
Your Answer:

Post as a guest

Your Name:

What's your source?

Earn Coins

Coins can be redeemed for fabulous gifts.

Not the answer you're looking for? Ask your own homework help question. Our experts will answer your question WITHIN MINUTES for Free.
Similar Homework Help Questions
  • Need to solve a,b and c inggriri 02%20-х pdt Consider the circuit shown below. 2.8,-50 R-100...

    Need to solve a,b and c inggriri 02%20-х pdt Consider the circuit shown below. 2.8,-50 R-100 kQ R,-20 kQ 1. Before the lab session, use the Kirchhoff's loop rule to manually calculate: a) the current through each resistor, b) the power supplied by each source, and c) the power dissipated in each resistor, and prepare your report and have it with you electronically at the time of your lab session.

  • Pre-lab EM-5 Ohm's Law and Kirchhoff's Rules Ohm's Law The resistance R of a device can...

    Pre-lab EM-5 Ohm's Law and Kirchhoff's Rules Ohm's Law The resistance R of a device can be determined by either directly measuring the resistance using an ohmmeter, or by measuring the current I through it and the voltage Vacross it, and then calculating R using Ohm's Law V R= (1) If the voltage across a resistor is 10V, and the current passing through it is 2.5 mA. of the resistor? What is the resistance R= Ω. Kirchhoffs Loop Rule Around...

  • How do I calculate the “calculated current flowing through each resistor using Kirchhoffs junction rule and...

    How do I calculate the “calculated current flowing through each resistor using Kirchhoffs junction rule and the measured values”? Please show your work. Thank you! Old Dominion University Physics 112 & 232 Lab 2. Use the EXTECH Digital M measure the individual resi "orange, black, brown" band Turn on the DMM and turn 2000 Ω position. 2 V 1.5 V Using wire leads with ab and alligator clip on opp banana plug end of a red wi 2) jack, and...

  • Analyze the following circuit with R1=1.0 kΩ R2=1.2 kΩ : a) Apply ohms law to find...

    Analyze the following circuit with R1=1.0 kΩ R2=1.2 kΩ : a) Apply ohms law to find I3. (Enter answer in mA with 3 decimals) b) Calculate the power dissipated in R2, Express your answer in mW with 3 decimals, c) Apply KCL at node x to determine I2. Enter your answer in mA with 3 decimals. d) Calculate the power supplied or absorbed by the voltage source. Express your answer as a signed mW value to 3 decimals. e) Apply...

  • ANALYSIS Use your experimental results to analyze the circuit in terms of Kirchhoff net current fow...

    ANALYSIS Use your experimental results to analyze the circuit in terms of Kirchhoff net current fow into and out of each of the four nodes, and determine whetheC s Rules. Consider supported by your data. Determine the net voltage drop around at leaston Sign determine whether or not your data supports the loop rule. (Pay close attosed loops convention.) In adition, verify ir Ohm's law is satisfied for at least three resistors and for the total reji >Q12: Why do...

  • Consider the circuit shown in the figure, where 1 = 21.7 V, 2 = 14.3 V,...

    Consider the circuit shown in the figure, where 1 = 21.7 V, 2 = 14.3 V, and R = 12.0 Ω. I'm not sure I understand Kirchhoff's Law and how to apply it to a problem like this. Consider the circuit shown in the figure, where & - 21.7 V, E- 14.3 V, and R 12.00. (Due to the nature of this problem, do not use rounded intermediate values in your calculations-including answers submitted in WebAssign.) 28.0 12.0 (a What...

  • ENGR 1181 Lab 3: Circuits Preparation Material Lab 3: Circuits Lab-Pre-lab Assignment Team Seat No. Name...

    ENGR 1181 Lab 3: Circuits Preparation Material Lab 3: Circuits Lab-Pre-lab Assignment Team Seat No. Name This is an individual assignment Solve the five problems below and hand it in at the beginning of the Circuits Lab. Show all your work Problem 1. Ohm's Law For the circuit below, calculate the value of the resistor R which would cause the current of 2.5 mA to flow in the circuit What voltage would you measure across the resistor? Show your calculations....

  • 8. Suppose the voltage output of a battery is 10.0V in a series circuit, and the...

    8. Suppose the voltage output of a battery is 10.0V in a series circuit, and the resistances are Ri=4.0012, R2=5.0092, and R3=6.002. a. Draw a diagram of the circuit, and determine the total resistance, the current through each resistor, the voltage drop across each resistor, and the power dissipated by each resistor and the source. 9. Suppose the voltage output of a battery is 10.0V in a parallel circuit, and the resistances are Ri=4.0012, R=5.0002, and R3=6.092. b. Draw a...

  • The RCL series circuit shown below (R=0.04 2, L=5.00uH) is in resonance with a source that...

    The RCL series circuit shown below (R=0.04 2, L=5.00uH) is in resonance with a source that has a frequency of 2000 Hz and a 12.0-V ims potential. a.) what is the capacitance of the circuit? for parts (6)-(f), the circuit is now driven with the same potential but with a frequency of 1700th. b.) Determine the phase angle Ø between the potential across the RCL circuit and the current induced by the AC source, which leads the current or potential?...

  • Consider the circuit as shown below. R-2 92 Use Kirchoff's rules to calculate (a) the current...

    Consider the circuit as shown below. R-2 92 Use Kirchoff's rules to calculate (a) the current through each resistor: Le. R1: [IR1] A R2: [IR2] A R3: [IR3] A R4: [IR4] A (b) the power supplied by each DC source;i.e. V1: [P1] Watts v2: [P2] Watts v3: [P3] Watts and (c) the power dissipated in each resistor; L.e. R1: [PR1] Watts R2: [PR2] Watts R3: [PR3] watts R4: [PR4] watts IR1 is equal to ?? Amps Answer: IR2 is equal...

ADVERTISEMENT
Free Homework Help App
Download From Google Play
Scan Your Homework
to Get Instant Free Answers
Need Online Homework Help?
Ask a Question
Get Answers For Free
Most questions answered within 3 hours.
ADVERTISEMENT
ADVERTISEMENT
ADVERTISEMENT